What is the constant rate???

What Is The Constant Rate???

Answers

Answer 1

Answer:

The constant rate is 4

Step-by-step explanation:

40 ÷ 10 = 4

80 ÷ 20 = 4

120 ÷ 30 = 4

160 ÷ 40 = 4

They all equal 4 so that is the constant rate.

Brainliest???


Related Questions

A standard clock has a 1-cm hour hand and a 2-cm minute hand. At 12 pm, they are both pointing in the same direction and the distance between the ends of the hands is 1 cm. What is the exact distance between the ends of the hands at 2 pm

Answers

Answer:

The distance between the hands is √(3)cm ≈ 1.73cm.

Step-by-step explanation:

In a standard clock, the angle between every number is 30°, therefore the angle between 12 and 2 will be 30° x 2 = 60°.

Looking at the diagram, to find c we can make use of our cosine formula

c² = a² + b² –2abCos(C°)

a = 2, b = 1 and C° = 60°

Therefore we have:

c² = 2² + 1² –2 x 2 x 1 x cos(60°) =

c² = 4 + 1 – 4 x 0.5 =

c² = 5 – 2 =

c² = 3

c = √(3) ≈ 1.73

Therefore, the distance between the hands is √(3)cm ≈ 1.73cm.

Jake has 30$ in his back account. he spent 53$ dollars at the fair. what is the balance in his back account after his trip to the fair ​

Answers

Wouldn't it be -23$ ? Since he had 30$ and take always 53$ so that would be 30-53= -23?

Paige makes bracelets and sells them online for $15 and $20. She buys all of her supplies in bulk. The table shows her sales for the last 3 months. During the 3-month period represented by the table, she ordered supplies twice. Each

Answers

Which table I don’t see it

What is the solution of the system? 2x + 5y = 14 x + 3y = 16

Answers

See attachment for math work and answer.

Which measure is not the area of a circle with radius 20 mm?

Answers

Answer:

C

Step-by-step explanation:

Answer:

thank you

Step-by-step explanation:

Find the area of a circle with a diameter of 6 cm

Answers

In terms of pi, 9pi.
Otherwise it would be 28.2735
The equation is pi r^2 so the radius is 3 and you just plug it in.

If u help me with this question you are amazing ❤️❤️

Answers

Answer:

the answer is A 90+0.25x=147.50

Step-by-step explanation:

if this is useful please make me brainliest

Xander takes a quiz worth 150 points. Each question is worth 30 points. Sketch a graph to show his score if he misses 1,2,3,4 or 5 questions.

Answers

Answer:

If he missed five questions he can get a zero

Step-by-step explanation:

-3, 6, -12, 24,.......... a9
What’s the missing term?

Answers

Answer:

-48 there is a pattern

PLEASE HELP STAT WILL MARK BRAINLIEST!

Mr. Lee has 67 rubber stamps. He wants to give the same number of stamps to each of his 8 students. If he gives away as many stamps as he can, how many stamps will be leftover? [remainder]

Answers

Answer:

3

Step-by-step explanation:

To give the same amount of stamps to each student you would divide 67 by 8. Dividing would give each students 8 stamps because you can only equally divide it to 64 (EX, 64/8=8). This leaves you with 3 stamps leftover, because we only used 64 equally and 67-64=3.

Answer:

He can give each of his students 8 stamps.

With 3 stamps left over.

Step-by-step explanation:

Hope this helps!

Solve the following system of linear equations by graphing:
-x+y = -7
4x + 4y = 20

Answers

I think it will be (-1,6)

during the first week of the Season Carly averaged 27 points per basketball game then she went on a scoring streak.​

Answers

Answer:

Part a: 7.5

Part B: It changes by 1.5

Step-by-step explanation:

PART A:

In the table you are suppose to find out how much it increases by so you subtract,

[tex]4.5 - 3 = 1.5[/tex]

[tex]6 - 4.5 = 1.5[/tex]

Since we know the difference is 1.5 you will add 6 by 1.5

[tex]6 + 1.5 = 7.5[/tex]

Therefore, the answer is 7.5

PART B: It changes by 1.5 because the difference between the increase in points is 1.5

simplify or solve the problem

4 ( 3x - 6 ) = 12

a) -3
b) 0
c) 3

Answers

X is equal to 3 which is (c)

1. If the total cost function for a product is C(x) = 200(0.02x + 6)3 dollars, where x represents the number of hundreds of units produced, producing how many units will minimize average cost?
2. A firm can produce only 3900 units per month. The monthly total cost is given by C(x) = 500 + 200x dollars, where x is the number produced. If the total venue is given by R(x) = 450x-1/100x^2 dollars, how many items, x, should the firm produce for maximum profit?
3. If the profit function for a product is P(x) = 3600x + 60x2 ? x3 ? 72,000 dollars, selling how many items, x, will produce a maximum profit?.

Answers

Answer:

a. The number of units which will minimize average cost is approximately 5,130 units.

b. The firm should produce 12,500 items, x, for maximum profit.

c. The number of items, x, that will produce a maximum profit is 60 items.

Step-by-step explanation:

Note: This question is not complete as there are some signs are omitted there. The complete question is therefore provided before answering the question as follows:

1. If the total cost function for a product is C(x) = 200(0.02x + 6)^3 dollars, where x represents the number of hundreds of units produced, producing how many units will minimize average cost?

2. A firm can produce only 3900 units per month. The monthly total cost is given by C(x) = 500 + 200x dollars, where x is the number produced. If the total venue is given by R(x) = 450x-1/100x^2 dollars, how many items, x, should the firm produce for maximum profit?

3. If the profit function for a product is P(x) = 3600x + 60x2 - x3 - 72,000 dollars, selling how many items, x, will produce a maximum profit?

The explanation to the answer is now given as follows:

1. If the total cost function for a product is C(x) = 200(0.02x + 6)3 dollars, where x represents the number of hundreds of units produced, producing how many units will minimize average cost?

Given;

C(x) = 200(0.02x + 6)^3 ……………………………………….. (1)

We first simplify (0.02x + 6)^3 as follows:

(0.02x + 6)^3 = (0.02x + 6)(0.02x + 6)(0.02x + 6)

First, we have:

(0.02x + 6)(0.02x + 6) = 0.004x^2 + 0.12x + 0.12x + 36 = 0.004x^2 + 0.24x + 36

Second, we have:

(0.02x + 6)^3 = 0.004x^2 + 0.24x + 36(0.02x + 6)

(0.02x + 6)^3 = 0.00008x^3 + 0.048x^2 + 7.20x + 0.0024x^2 + 1.44x + 216

(0.02x + 6)^3 = 0.00008x^3 + 0.048x^2 + 0.0024x^2 + 7.20x + 1.44x + 216

(0.02x + 6)^3 = 0.00008x^3 + 0.0504x^2 + 8.64x + 216

Therefore, we have:

C(x) = 200(0.02x + 6)^3 = 200(0.00008x^3 + 0.0504x^2 + 8.64x + 216)

C(x) = 0.016x^3 + 10.08x^2 + 1,728x + 43,200

Therefore, the average cost (AC) can be calculated as follows:

AC(x) = C(x) / x = (0.016x^3 + 10.08x^2 + 1,728x + 43,200) / x

AC(x) = (0.016x^3 + 10.08x^2 + 1,728x + 43,200)x^(-1)

AC(x) = 0.016x^2 + 10.08x + 1,728 + 43,200x^(-1) …………………………. (2)

Taking the derivative of equation (2) with respect to x, equating to 0 and solve for x, we have:

0.032x + 10.08 - (43,300 / x^2) = 0

0.032x + 10.08 = 43,300 / x^2

X^2 * 0.32x = 43,300 – 10.08

0.32x^3 = 43,189.92

x^3 = 43,189.92 / 0.32

x^3 = 134,968.50

x = 134,968.50^(1/3)

x = 51.30

Since it is stated in the question that x represents the number of hundreds of units produced, we simply multiply by 100 as follows:

x = 51.30 * 100 = 5,130

Therefore, the number of units which will minimize average cost is approximately 5,130 units.

2. A firm can produce only 3900 units per month. The monthly total cost is given by C(x) = 500 + 200x dollars, where x is the number produced. If the total revenue is given by R(x) = 450x-1/100x^2 dollars, how many items, x, should the firm produce for maximum profit?

P(x) = R(x) - C(x) ……………. (3)

Where;

P(x) = Profit = ?

R(x) = 450x-1/100x^2

C(x) = 500 + 200x

Substituting the equations into equation (3), we have:

P(x) = 450x - 1/100x^2 - (500 + 200x)

P(x) = 450x - 0.01x^2 - 500 - 200x

P(x) = 450x - 200x - 0.01x^2 - 500

P(x) = 250x - 0.01x^2 – 500 …………………………………. (4)

Taking the derivative of equation (4) with respect to x, equating to 0 and solve for x, we have:

250 - 0.02x = 0

250 = 0.02x

x = 250 / 0.02

x = 12,500 items

Therefore, the firm should produce 12,500 items, x, for maximum profit.

3. If the profit function for a product is P(x) = 3600x + 60x2 – x^3 - 72,000 dollars, selling how many items, x, will produce a maximum profit?

Given;

P(x) = 3600x + 60x2 – x^3 - 72,000 …………………………. (5)

Taking the derivative of equation (5) with respect to x, equating to 0 and solve for x, we have:

3600 + 120x - 3x^2 = 0

Divide through by 3, we have:

1200 + 40x – x^2 = 0

1200 + 60x – 20x – x^2 = 0

60(20 + x) – x(20 + x) = 0

(60 – x)(20 + x) = 0

Therefore,

x = 60, or x = - 20

The negative value of x (i.e. x = - 20) will be will be ignored because it has no economic significance. Therefore, the number of items, x, that will produce a maximum profit is 60 items.

Molly has 9 lollipops. Her friend has 3 times as many. How many lollipops does Molly's friend have?

27
29
31
33

Answers

Answer:

27

Step-by-step explanation:

Since 9 is the amount of lollipop Molly has and her friend has 3 times more; which is 9x3

Can some one help please!!!!!!!

Answers

Answer:

yes it's easy

Step-by-step explanation:

y=-2x

it may help you to understand.

Jason practiced making soccer goals and kept her track of the results. He missed 3 goals but made 9 in one practice session. What can Jason predict regarding goal made if his next practice session has 24 tries?

Answers

He can predict to make 8 out of 24

PLEASE HELP! IMAGE ATTACHED

Answers

The answer is c
Your welcome

Judy had a bag with 12 DVDs, 12 marbles, 11 books, and one orange. What percentage of the items in the bag are DVDs

Answers

Answer:

34.28 percent of the items in the bag are DVDs

Step-by-step explanation:

Given that he had following items

DVDs 12

MArbles 12

Books 11

Total items are: 11+12+12 = 35

In order to find the percentage of DVDs we have to divide the quantity of DVDs by total and multiply by 100

So,

Percentage of DVDs is:

[tex]=\frac{12}{35} *100\\= 0.3428*100\\= 34.28\%[/tex]

Hence,

34.28 percent of the items in the bag are DVDs

Helppppppppppppppppppp pleaseeeeeeeeee

Answers

Answer:

1.5 goes in between the 0 and the first line (meaning 1) and 0.5 goes in between the left of the zero and the line before the zero


Two towns that are 31.5 km apart are 3 cm apart on a map. What is the scale of the map?
A. 1 cm = 29.5 km
B. 1 cm = 9.5 km
C. 1 cm = 10.5 km
D. 1 cm = 15.5 km

Answers

Answer:

3 cm on the map represents 31.5 km in reality.

3cm : 31.5 km

1cm :  31.5/3 km

1cm : 10.5km

So the scale on the map is 1cm represents 10.5 km.

Which is true about the solution to the system of inequalities shown

Answers

pls provide the problem

Answer:

look below for question

Step-by-step explanation:

Which is true about the solution to the system of inequalities shown?

y > 3x + 1

y < 3x – 3  

Only values that satisfy y > 3x + 1 are solutions.

Only values that satisfy y < 3x – 3 are solutions.

Values that satisfy either y > 3x + 1 or y < 3x – 3  are solutions.

There are no solutions.

I have 8 Twix bar that I want to share with 6 students how much of the Twix bar will each student get

Answers

Answer:

1.3

Step-by-step explanation:

8 divided by 6 = 1.3

BRAINLIEST PLEASE

Answer:

1.3

Step-by-step explanation:

Hope this helped have an amazing day!

If 1 and 2 are vertical angles, <2 and <3 are complementary angles,
and m23 = 56°, find m1

Answers

Answer:

hehdhrhth

Step-by-step explanation:

thrhrgefwy3ihrgej eff wr3 utf egtje creek it g

If f x) = [x – 3], find f (5.6 ). %3D

Answers

Answer:

f(5.6) = 2.6

General Formulas and Concepts:

Pre-Algebra

Order of Operations: BPEMDAS

Brackets Parenthesis Exponents Multiplication Division Addition Subtraction Left to Right

Algebra I

Function Notation

Step-by-step explanation:

Step 1: Define

f(x) = [x - 3]

f(5.6) is x = 5.6

Step 2: Evaluate

Substitute in x:                    f(5.6) = [5.6 - 3]Subtract:                              f(5.6) = 2.6

What is 1 3/4 hours in minutes?

Answers

Answer: 105

Step-by-step explanation:


Question 2
What does the flow of electricity require?
A
conductors and an open circuit
B
insulators and an open circuit
C
conductors and a closed circuit
D
insulators and a closed circuit

Answers

Answer:

a

Step-by-step explanation:

what is the greatest common factor of 3/5

Answers

Answer:

15

Step-by-step explanation:

3 × 5 = 15

Hope this helped <3

the answer is 3/5.. like i’m not joking

I need help, please
it is do tomorrow

Answers

Answer:

#2) 30% of 400 is 120 and 120/6 = 2 so it took him 2 hours

#3) 2.5/3.2 = .78 and 1/1.2 =.83 so buying the bigger one you are paying less

hey get you points
hahahaha

Answers

Thank youuuuuuuuuuuuuuuuuu
thank you so muchhhh!
Other Questions
Carmen has taken out a loan for $800 to buy a car. She plans to pay back the loan at a rate of $40 per month. Ramona has borrowed $500 to buy a car, which she plans to pay back at a rate of $20 per month.How long will it take Ramona to pay back her loan? !!!!!!!!!!!!!!!!!!!!!!!!!!!!!!!!!!!!!!!!!!!!!!!!!!!!!!!!!!!!!!!!!!!!!!!!!!!!!!!!!!!!!!!!!!!!!!!!!!!!!1 The perimeter of a square Is 176 centimeters. If the square is dilated by a scale factor of 0.75, what is the length of each side of the new square? Lucy has a budget for school stationery of 33, but has already spent 22.47 on books and folders. Let p represent the amount that Lucy can spend on other stationery. Write an inequality that shows how much she can spend on other stationery, and solve for p. Find the slope of the line that passes through the points A(5,-1) and B (3, -1). which word does not belong?glass, piece,let,vase ignore how i drew on the screen, also ill give brainliest PLEASE HURRY!!Mildred classifies the following system of equations.y = 4x + 3y = 6x + 1Select Correct or Not correct for each statement.Statements: The system is consistent because the slopes and the y-intercepts are different. The system is independent because the slopes and the y-intercepts are different. Which lines from the poem make up the actual "page" that the speaker writes for his instructor? How do they differ from the rest of the poem? Provide details from the text that support your response. Find the slope between the following points (2,5) and (-3,-9) Write an equation of the line passing through point (7,3) that is parallel to line 4x-7y=9 1. What factors can change the attractive force between the nucleus of an atom andits valence electrons? How many men did it take to capture Antigone?A:2B:5C:1D:7 You ride the subway 45 times per month. Find your monthly cost without purchasing a monthly pass. Then determine whether you should buy the monthly pass. X: -3, 2, 7, 12Y: 0, 2, 4, 6Whats the slope ? Psychologists often talk of the nature-nurture controversy. Which of theseconcepts supports the "nurture" over the "nature" part?O A. Freudian conceptsO B. James-LangeOC. Tabula rasaOD. Biological origins Drag the tiles to the correct boxes to complete the palrs.Match the types of letters with their descriptions.letter of interestcover letterthank you letterletter of recommendationdiscusses how you meet job requirements>reiterates why you're the best candidate for the job>tells more about your character and who you areexpresses your interest in working for a company What is the length of line segment BC ILL GIVE BRAINLIEST PLS HELP What is x/2 + 8 < 10. x < _____